jueves, 3 de junio de 2010

N4 lista corta

Encuentra todas las ternas (a, m, n) de enteros positivos tales que a^m+1 divida a (a+1)^n.

10 comentarios:

Flavio dijo...

es a^(m+1) o (a^m)+1?

Georges dijo...

Haber esto es lo que llevo, claramente si a=1 y n y m son enteros positivos es solución, luego si m=1 y a y n son enteros positivos tambien es solución.Entonces supongamos que a y m son mayores que uno, entonces a^m+1>2.
Ahora supongamos que m es par.Como m es par entonces a+1 divide a (a^m)-1 (a^m congruente con (-1)^m congruente con 1). Por lo tanto si un entero d, divide a a+1 entonces divide a (a^m)-1, y si d tambien divide a (a^m)+1 entonces d divide a (a^m)+1-((a^m)-1)=2Por lo tanto mcd((a^m)+1,a+1)=1 o 2. Si es uno entonces ((a^m)+1,(a+1)^n)= 1 y entonces el primer número no divide al segundo por ser primos relativos.Si es dos entonces claramente (a^m)+1 tiene que ser una potencia de dos pero como ya sabemos que (a^m)+1>2 entonces (a^m)+1 es congruente con cero mod 4, pero un cuadrado (m es par) nunca puede ser congruente con 3 mod 4.Por lo tanto si a y n son mayores que uno entonces m tiene que ser impar.

El caso que m es impar lo voy a subir en un rato (cuando me salga) jaja..

Unknown dijo...

Es lo segundo Flavio

Unknown dijo...

Si, yo también ya tengo que m es impar. No he leído tu solución de esto Georges, no la quiero leer porque no lo he terminado, pero por la extensión que lo que escribiste creo que hice algo más corto.

DANIELIMO dijo...

llegue a que m es impar

Flavio dijo...

Pues segun yo ya me salio y la unica solucion no trivial es m=3,a=2,n>=2, pero no estoy muy seguro de que todo este bien, no se si conclui bien o me falto ver algo:
como ya varios vieron
se tiene m impar o a=1 suponemos que a>1.
Luego si d|m entonces a^d+1|a^m+1, ya que podemos factorizar suma de potencias impares
(a la m/d).
ahora veamos que no hay solucion con m un primo impar, entonces, no habra solucion para
cualquier m impar mayor a 1 por que tendra un divisor primo p, entonces
a^p+1 | a^m+1 entonces a^p+1|(a+1)^n, entonces no puede haber solucion ya que para primos
impares no habra.
sea m primo impar entonces, ahora le llamare p.
Supongamos que existe a tal que
a^p+1 | (a+1)^n para nuestro p
pero a^p+1 = (a+1)*(a^(p-1)-a^(p-2)+...+1), entonces
(a^(p-1)-a^(p-2)+...+1) | (a+1)^(n-1)
pero supongamos que un primo q divide a (a^(p-1)-a^(p-2)+...+1), entonces, q divide
a a+1, entonces a=-1 mod q, luego (a^(p-1)-a^(p-2)+...+1) = p mod q. Entonces q divide a
(a^(p-1)-a^(p-2)+...+1) y entonces q divide a p. Entonces p=q, ya que q y p son primos.
entonces (a^(p-1)-a^(p-2)+...+1)= p^r para algun entero r, ya que todos los primos que
dividen a q deben ser iguales a p.
Luego como p^r|a+1 entonces
a=pk-1, para algun entero k,
entonces (a^(p-1)-a^(p-2)+...+1) = (a^p+1)/(a+1)= ((pk-1)^p+1)/(pk) entonces
(a^(p-1)-a^(p-2)+...+1) = ((pk)^p+(pC1)*(pk)^(p-1)*(-1)+...+(pC(p-2))*(pk)^2*(-1)^(p-2)+(pC(p-1))*pk*(-1)^(p-1)+(-1)^p+1)/pk
pero todos los terminos son multiplos de p^3*k^2, excepto por los ultimos tres terminos.Pero los ultimos dos son 1 y -1 y se cancelan
entonces al dividir entre pk queda un multiplo de p^2 mas (pC(p-1))*pk*(-1)^(p-1)/pk = p*(-1)^(p-1) entonces
p^2 no divide a (a^(p-1)-a^(p-2)+...+1) entonces como (a^(p-1)-a^(p-2)+...+1)=p^r para algun entero r,
entonces (a^(p-1)-a^(p-2)+...+1) = p, entonces (a^p+1)/(a+1)=p
entonces
(a^p+1)/(a+1)-1=p-1 entonces (a^p+1-(a+1))/(a+1)=p-1, entonces (a^p-a)/(a+1)=p-1
pero p | a^p-a por teorema de fermat, pero p|p-1, entonces p|a+1 , ademas entonces
(a^p-a)/(a+1)=p-1=a*(a^(p-1)-1)/(a+1)pero como a+1 y a son primos relativos, entonces
a+1 | a^p-1, luego (a^(p-1)-1)/(a+1) es un entero, entonces a|p-1 , y ademas teniamos que p|a+1
entonces como son positivos todos, entonces a<=p-1, y p<=a+1 entonces p-1<=a, entonces p-1=a
entonces veamos para que primos p se cumple que
(p-1)^p+1|p^n y ademas tenemos que ((p-1)^p+1)/(p)=p, entonces (p-1)^p+1=p^2
<=> (p-1)^p=p^2-1=(p-1)*(p+1)
<=> (p-1)^(p-1)=p+1 => p-1|p+1 pero p-1|p-1 entonces p-1|p+1-(p-1)=2
entonces p-1=2 o 1 pero teniamos p impar, entonces p-1=2, entonces p=3. Entonces cualquier primo que divida a m
debe ser 3, bueno cuando m es impar
entonces m=3^r para r entero no negativo. Ademas demostre que a=p-1, entonces a=2
entonces hay que ver cuando
2^(3^r)+1|3^n, entonces queremos que 2^(3^r)+1=3^k para un entero no negativo k
pero si m>1, las unicas potencias a diferencia 1 son 8 y 9 por catalan.
entonces 2^(3^r)=8, entonces r=1, m=3, entonces m=3 o m=1.
pero si m=1, entonces queremos que
a+1|(a+1)^n, entonces todas las a y n son soluciones.
si m=3, ya vimos que entonces a=2,
y luego 2^3+1|3^n, entonces 9|3^n, entonces n>=2, a=2, m=3 es solucion, y por como se construyo, son las unicas soluciones.
a bueno y las de a=1.

DANIELIMO dijo...

solucion(a medias):
si p primo divide a a^m +1, a congruentecon a^m congruentecon -1(mod p) por lo tanto m es impar.
si q primo divide a m, a^q +1 divide a a^m +1 que divide a (a+1)^n por lo tanto X= (a^q +1)/(a+1) divide a (a+1)^(n-1). supongamos que un r primo distinto a q divide a X entonces a congruentecon -1 (mod r), y entonces 0 congruentecon X congruentecon q (mod r) lo que es una contradicción
por lo tanto X es una potencia de p

y ya encontre un error, asi q hasta aqui he llegado. Voy a checarlo a ver si encuentro como acabar

Unknown dijo...

Yo tengo eso de que m es impar y que si p primo divide a (a^{m}+1)/(a+1) entonces p divide a m. En resumen sólo tengo eso de lo que creo que pueda servir, no se me ha ocurrido nada nuevo...

Unknown dijo...

Por fin tengo una solución para este problema!!! La puse en un nuevo post por si a alguien le interesa verla...

Unknown dijo...

Teorema de zsigmondy

Publicar un comentario